Feladat: 2015. évi Nemzetközi Matematika Diákolimpia 23. feladata Korcsoport: 18- Nehézségi fok: nehéz
Megoldó(k):  Fehér Zsombor 
Füzet: 2015/október, 393 - 395. oldal  PDF  |  MathML 
Témakör(ök): Nemzetközi Matematikai Diákolimpia, Egyenlőtlenségek, Számhalmazok
Hivatkozás(ok):Feladatok: 2015/szeptember: 2015. évi Nemzetközi Matematika Diákolimpia 23. feladata

A szöveg csak Firefox böngészőben jelenik meg helyesen. Használja a fenti PDF file-ra mutató link-et a letöltésre.

 
Fehér Zsombor megoldása. Legyen cj=aj+j. Ekkor az (i) feltétel azt mondja ki, hogy j+1cjj+2015, a (ii) feltétel pedig azt, hogy a cj számok mind különbözőek.
Megmutatjuk, hogy a c1,c2,... sorozat véges sok kivétellel minden pozitív egész számot felvesz. Tegyük fel ugyanis, hogy legalább 2016-ot nem vesz fel, és legyen t egy olyan pozitív egész, ami nagyobb ennél a 2016 számnál. Ekkor az (i) feltétel alapján a {c1,c2,...,ct} halmaz minden eleme az [1,t+2015] intervallumba esik, és mivel (ii) szerint t különböző elemről van szó, ezért ebből az intervallumból {c1,c2,...,ct} éppen 2015 pozitív egész számot nem vesz fel. Azonban feltevésünk szerint az ennél bővebb {c1,c2,...} halmaz legalább 2016 darab t-nél kisebb pozitív egész számot nem vesz fel, ami pedig ellentmondás.
A feladatnak megfelelő b számot válasszuk meg annyinak, amennyi a c1,c2,... sorozat által fel nem vett pozitív egészek száma, N pedig legyen egy olyan szám, ami nagyobb ennél a b darab kimaradó számnál. A fenti gondolatmenetből az is látható, hogy b2015. Az m, n pozitív egészekre a továbbiakban feltesszük, hogy Nm<n.
A feladatunk lényegében az, hogy egy aj kifejezést megfelelő korlátok közé szorítsunk, ami nyilván ugyanaz, mint cj megfelelő korlátok közé szorítása. Tudjuk, hogy {cm+1,...,cn} minden eleme az [m+2,n+2015] intervallumba esik, és mivel ezen intervallum n-m+2014 egész számából n-m van az előző halmazban, ezért 2014 egész szám marad ki. Vizsgáljuk meg közelebbről ezt a 2014 számot: ki fog derülni, hogy közülük b-1 darab az [m+2,n+2015] intervallum ,,elején'', 2015-b pedig a ,,végén'' helyezkedik el.
Mivel a {c1,c2,...} halmaz b darab egész számot nem vesz fel az [1,) intervallumból, ezért a {cm+1,cm+2,...} halmaz b+m számot nem vesz fel [1,)-ből. Ez cj>j alapján azt jelenti, hogy a {cm+1,cm+2,...} halmaz b-1 számot nem vesz fel [m+2,)-ből. Mivel azonban m+2>N, ezért ezen b-1 számot is felveszi valahol a c1,c2,... sorozat, csak még cm+1 előtt. Így ez a b-1 szám mindegyike olyan ck, melyre km, így ckk+2015m+2015 alapján ezek a számok mind az [m+2,m+2015] intervallumba esnek.
Tehát azon 2014 egész közül, melyek az [m+2,n+2015] intervallumban benne vannak, de a {cm+1,...,cn} halmazban nem, b-1 darab az {c1,...,cm} halmazban van, a maradék 2015-b darab pedig szükségképpen a {cn+1,...} halmazban. Ezen 2015-b szám mindegyike legalább n+2, így ezek az [n+2,n+2015] intervallumba esnek.
Ezen a ponton álljunk meg egy pillanatra, és vegyük észre, hogy a feladat megoldásával lényegében készen vagyunk. Csak az alapján, hogy a 2014 kimaradó szám valahol az [m+2,n+2015] intervallumban van, még nem tudnánk pontos becslést mondani, hiszen m,n-et kicsivel megváltoztatva az egyik kimaradó szám szabadon ,,átugorhatna'' az intervallum elejéről a végére, ezzel nagy (n-m nagyságrendű) változást eredményezve. De azáltal, hogy a 2014 kimaradó szám közül mindig b-1 van az intervallum elején, és 2015-b a végén (ahol a b egy univerzális paramétere a sorozatnak!), ilyen ugrások nem történhetnek meg, csak az intervallum szélein lévő rövid (2014 hosszú) intervallumok belsejében mozoghatnak a kimaradó számok. Így lehetséges az, hogy m, n-től független, 10072 nagyságrendű becslést fogunk tudni mondani.
Nem maradt más hátra, minthogy kiszámoljuk a 2014 kimaradó szám összegének lehetséges legkisebb és legnagyobb értékét, majd ezt visszavezessük a feladatbeli összegre. Tudjuk, hogy a 2014 szám felbontható valahogy egy b-1 és egy 2015-b elemű csoportra, melyek elemei rendre az [m+2,m+2015], illetve az [n+2,n+2015] intervallumból valók. (Előfordulhat, hogy ez a két intervallum átfedi egymást, de ez nem okoz gondot.) Mivel a számok különbözőek, ezért a 2014 szám összege legalább
((m+2)+(m+3)++(m+b))++((n+2)+(n+3)++(n+2016-b))==(b-1)(2m+b+2)2+(2015-b)(2n+2018-b)2=hmin,


legfeljebb pedig
((m+2017-b)++(m+2015))+((n+b+1)++(n+2015))==(b-1)(2m+4032-b)2+(2015-b)(2n+2016+b)2=hmax.



Ha H jelöli az előbbi 2014 kimaradó szám összegét, akkor a feladatban szereplő összeg így írható:
j=m+1n(aj-b)=j=m+1n(cj-j-b)=i=m+2n+2015i-H-j=m+1nj-j=m+1nb==(n+2014-m)(n+m+2017)2-H-(n-m)(n+m+1)2-b(n-m).


A hminHhmax becslést alkalmazva, a kifejezések egyszerűsítése után végül a következőt kapjuk:
b2-2016b+2015j=m+1n(aj-b)-b2+2016b-2015.
Így tehát valóban,
|j=m+1n(aj-b)|-b2+2016b-2015=10072-(b-1008)210072.